LSAT and Law School Admissions Forum

Get expert LSAT preparation and law school admissions advice from PowerScore Test Preparation.

 Administrator
PowerScore Staff
  • PowerScore Staff
  • Posts: 8950
  • Joined: Feb 02, 2011
|
#22712
Complete Question Explanation

Must Be True. The correct answer choice is (E)

The stimulus points out that several cosmetic firms are committed to developing means of testing using human cell cultures, in order to reduce the need for tests on live animals.

As a point of interest, since the stimulus implies that these cosmetic firms believe their commitment is justified and a good idea, and the question uses the word "support," a great many people will incorrectly answer this question simply because they attempt to strengthen the stimulus. Of course, you should not have made that error since you know that every question in this section is Must Be True. However, on the actual test you should notice that when the question is about support, you need to be careful to note whether the stimulus should support a choice, or a choice should support the stimulus. Frequently, when the LSAT test writers introduce a Must Be True question using the word "support," the stimulus will consist of statements that might be in need of support, and there will be strengthening answer choices, but those choices are always wrong because they do not accomplish the question task. The correct answer might not be perfect, but will follow after the information in the stimulus, rather than seem like support for the stimulus.

Answer choice (A): This is a must be true question, and the correct answer choice should not bring in outside information, so this choice is incorrect.

Answer choice (B): Since our goal is neither to weaken nor to strengthen, answer choices, such as this one, that seem to attack or support the stimulus are incorrect.

Answer choice (C): Since you were not supposed to strengthen the idea that it is a good idea to use human cell cultures, this answer choice which brings in new, potentially supportive information is incorrect.

Answer choice (D): This choice might support the idea that cosmetic firms should use human cell cultures; however, you are answering a Must Be True question, so this response is incorrect because it is unsupported by the stimulus. Also, the stimulus states that there will be fewer tests on animals, not fewer overall tests.

Answer choice (E): This is the correct answer choice, even though it is definitely imperfect. You can assume that a manager of a firm usually believes that his plan is a good idea. Since the reason for support of the plan is that the use of human cell cultures reduces testing on animals, we know that managers probably do think it is better to avoid testing on animals. Even though the stimulus did not mention managers, this choice is something that follows from the stimulus, and the other answer choices are either contrary to the stimulus, or engage in a misguided attempt to support the stimulus. Understanding the question stem and the proper flow of information is essential.
 Blueballoon5%
  • Posts: 156
  • Joined: Jul 13, 2015
|
#37576
Hello! I have a question about the answer key explanation (not necessarily the question itself). The answer key explains the difference between "must be true" and "strengthen" question stem. It states, "However, on the actual test you should notice that when the question is about support, you need to be careful to note whether the stimulus should support a choice, or a choice should support the stimulus."

My question: Which of the two ("must be true" and "strengthen") is "the stimulus should support a choice" and "a choice should support the stimulus"? My guess is that a "must be true" question stem = "the stimulus should support a choice," and a "strengthen" question stem = "a choice should support the stimulus." I think I might be right, but I would really appreciate a confirmation. Thank you so much!
User avatar
 Dave Killoran
PowerScore Staff
  • PowerScore Staff
  • Posts: 5972
  • Joined: Mar 25, 2011
|
#37589
Hi Blue,

You are correct! This all goes back the four question Families and the flow of information therein, if you want a reference for further reading.

Thanks!
 Blueballoon5%
  • Posts: 156
  • Joined: Jul 13, 2015
|
#37592
Thanks again Dave! :-D
 guilhermeb
  • Posts: 4
  • Joined: Jun 15, 2018
|
#46823
Could you explain again the difference between answer C and the correct answer E. In what ways does C attempt to strengthen the question and is wrong? And in what ways does E meets the necessary standards required by the question without assuming too much? I guess my question is "to what extent can you assume in the MUST BE TRUE questions?"
 Adam Tyson
PowerScore Staff
  • PowerScore Staff
  • Posts: 5387
  • Joined: Apr 14, 2011
|
#46836
While we treat this question as being a part of our Must Be True, or "Prove" family, guilhermeb, take note that the question stem actually sets up a slightly lower standard than "must be true": this is a Most Strongly Supports" question! In other words, the stimulus doesn't have to absolutely prove the answer choice is necessary, but instead it must provide substantial support for it. Think of this as a strengthen question in reverse, in that the stimulus will strengthen the correct answer. Or, if you prefer, think of it as a weaker version of a Must Be True, where the correct answer is the one that is most reasonably inferred based on the stimulus and with no outside information, even if it isn't completely necessary.

Answer C is an incorrect answer because the stimulus gives us no reason to believe anything about what the financial consultants believe. In fact, there is no reason to believe that any financial consultants were consulted! This answer, if it was true, would support the decision made by these firms to switch to testing on human cell cultures, and so in that sense it is a strengthen answer, but is not supported by the stimulus.

Answer E is supported by the stimulus, at least to some reasonable extent. The evidence is that using these cultures reduces the need for animal testing, and the firms are committed to making the switch. If that's true, then at least someone in charge must have determined that less animal testing was a good idea! It can't just be the customers who feel that way, or the shareholders, because even if pressure came from these outside groups, someone within the firm - a decision maker - has to have decided it was a good idea to listen to those folks. "Managers" is a reasonable catch-all to describe "the people in charge" who made that commitment to switch.

In short: when the stem asks about which answer is most strongly supported, we can accept a lower standard of proof, but the answer must still rely solely on what we read in the stimulus with no outside help and no unwarranted assumptions.
 whardy21
  • Posts: 48
  • Joined: Sep 30, 2018
|
#63923
My choices were down to E an D. I chose D. When we complete must be true/most strongly supported questions, is there a slight difference as far as certainty. The question stem is a most strongly be supported. I agree that E is correct if it is a strongly supported question. When I use the fact test, I don't see anything in the stimulus saying its better for their firms not to perform tests on live animals. If the question stem said which one of the following can be inferred, I would categorize E as an instant loser. Please give me a hand. Is there a slight degree of certainty between Must be True and Most Strongly Supported?
 Adam Tyson
PowerScore Staff
  • PowerScore Staff
  • Posts: 5387
  • Joined: Apr 14, 2011
|
#63947
Absolutely, whardy21, and I think of it as similar to the difference between a Strengthen question and a Justify the Conclusion question. In Strengthen, the correct answer will help the argument, but it might only help a little bit, while in Justify the correct answer will absolutely prove the conclusion of the argument.

With a true Must Be True question (which of the following can be inferred, or follows logically, or must be true, etc.) the standard of proof is very high, like Justify - the correct answer should be conclusively proven by the facts given in the stimulus with no additional outside information. For Most Strongly Supported questions, the stimulus should essentially strengthen the correct answer. I like to say that the correct answer to a MSS question is the one that is the most reasonable inference we could make under the circumstances, with little to no additional outside help. The standard is lower here than for a MBT question, although it could still be a strong answer that would pass the MBT test (just as a Strengthen answer might be good enough the Justify the Conclusion).

Answer D is not supported by the stimulus at all - it is, in my view, a completely unreasonable inference to make. What evidence is there that anyone thinks "fewer tests" will be needed? Maybe we will need to triple the number of tests, but it's still better because the tests will cost one-tenth what the old ones did? Because public perception will be improved? Because the tests will give more reliable results and lead to substantial product improvements, greater safety, fewer lawsuits, etc.? We also cannot infer anything about what researchers think, but only what decision-makers think (and that's why "managers" in answer E is a much better group of people to focus on). Maybe researchers are also decision-makers, maybe not.

Good analysis of the different standards, but I would argue that there IS evidence in the stimulus to support the claim of "better." The firms are arguing in favor of doing something new, and that, by itself, means that the thing they want to do is, at least in their estimation, better than whatever they are currently doing. It would make no sense at all to suggest that anyone ever argues that we should do something worse, because arguments in favor of something are always about gaining some benefit, getting something better. They don't have to explicitly say "this is better" in the stimulus for it to pass the Fact Test, so long as the answer is based solely on what we did read and without making "assumptions that are by commonsense standards implausible, superfluous, or incompatible with the passage." (quoting from the section instructions)
User avatar
 ashpine17
  • Posts: 331
  • Joined: Apr 06, 2021
|
#92314
So I was stuck between D and E and I had issues with E because I thought nothing in the stimulus talked about what MANAGERS BELIEVED. Am I supposed to let this go because this is a MSS and not a MBT? As for what the managers believed, is that supposed to be supported by the first sentence: these cosmetics firms are COMMITTED to the development of tests that use HUMAN cells?
 Robert Carroll
PowerScore Staff
  • PowerScore Staff
  • Posts: 1819
  • Joined: Dec 06, 2013
|
#92903
ashpine,

Cosmetics firms are not agents with their own will. The beliefs and actions of actual human beings are ascribed to the firm. So when the firms "are committed" to something, there has to be someone at the firms actually doing something. Answer choice (E) just uses the generic word "managers" for these people.

Robert Carroll

Get the most out of your LSAT Prep Plus subscription.

Analyze and track your performance with our Testing and Analytics Package.